Lp - Rom

Her kan du stille spørsmål vedrørende problemer og oppgaver i matematikk på høyskolenivå. Alle som har kunnskapen er velkommen med et svar. Men, ikke forvent at admin i matematikk.net er spesielt aktive her.

Moderatorer: Vektormannen, espen180, Aleks855, Solar Plexsus, Gustav, Nebuchadnezzar, Janhaa

Svar
madfro

Hei, er det noen som kan gi en forklaring på denne oppgaven?

Suppose that [tex]\Omega[/tex] is bounded and that [tex]1 \leq p \leq q \leq \infty[/tex].
Prove that [tex]L^q(\Omega)\subset L^p(\Omega)[/tex]. (Hint: Use Hölder's Inequality).
Aleks855
Rasch
Rasch
Innlegg: 6859
Registrert: 19/03-2011 15:19
Sted: Trondheim
Kontakt:

Hvis $f\in L^q(\Omega)$ ønsker du å vise at $\int_{\Omega} \lvert f\rvert^p\,d\mu$ er endelig. Hölders ulikhet kan hjelpe til med å gi en øvre grense på integralet.
Bilde
madfro

Ok, da er jeg et steg videre.

Sliter litt med å se hvordan Hölder's ulikhet skal brukes, da jeg ikke vet om sammenhengen [tex]\frac{1}{p} + \frac{1}{q} = 1[/tex] er oppfyllt. Her det noen vei rundt dette? Kan man for eksempel utnytte at man vet at [tex]q \geq p[/tex], slik at [tex]p = q - r, r \geq 0[/tex] og omformulere Hölder's ulikhet fra den originale formen?
Aleks855
Rasch
Rasch
Innlegg: 6859
Registrert: 19/03-2011 15:19
Sted: Trondheim
Kontakt:

Ja, du er inne på noe. Tenk at $\frac{1}{p}=\frac{1}{q}+\frac{1}{r}$ for $r > 0$.

Kanskje et litt tynt hint, men jeg tror et videre hint kan være i overkant stort.
Bilde
Gustav
Tyrann
Tyrann
Innlegg: 4560
Registrert: 12/12-2008 12:44

Noen flere hint

$\int_{\Omega} |fg|\leq (\int_{\Omega}|f|^r)^{\frac{1}{r}} (\int_{\Omega}|g|^s)^{\frac{1}{s}} $.
Sett $g=1$. Siden $\Omega$ er begrenset er dermed $\int_{\Omega}d\mu<\infty$, (i det minste for endelig mål $\mu$)
madfro
Noether
Noether
Innlegg: 43
Registrert: 08/03-2016 13:19

Da tror jeg at jeg forstår beviset :)

Takk for hint!

Er ny på området, og leser på egenhånd, så det kan nok bli flere spørsmål etter hvert :)
Gustav
Tyrann
Tyrann
Innlegg: 4560
Registrert: 12/12-2008 12:44

plutarco skrev:Noen flere hint

$\int_{\Omega} |fg|\leq (\int_{\Omega}|f|^r)^{\frac{1}{r}} (\int_{\Omega}|g|^s)^{\frac{1}{s}} $.
Sett $g=1$. Siden $\Omega$ er begrenset er dermed $\int_{\Omega}d\mu<\infty$, (i det minste for endelig mål $\mu$)
For ordens skyld, her er resten av beviset:

La $f\to f^p$, og $r=\frac{q}{p}\geq 1$. Da blir $\int_{\Omega} |f^p|\leq (\int_{\Omega}|f|^q)^{\frac{1}{r}} \mu (\Omega)^{\frac{1}{s}}<\infty $
Svar